Please help I need the code asap

Please Help I Need The Code Asap

Answers

Answer 1

The values based on the exponent will be:

0.024

5670

41952

0.005

73

0.34

900

6

How to calculate the values

The exponent is the number of times that a number is multiplied by itself. It should be noted that the power is an expression which shows the multiplication for the same number. For example, in 6⁴ , 4 is the exponent and 6⁴ is called 6 raise to the power of 4.

1) 2.4 × 10^-2 = 0.024

2) 5.67 x 10^3 = 5670

3) 4.1952 x 10^4 = 41952

4) 5 × 10^-3 = 0.005

5) 7.3 × 10^1 = 73

6) 3.4 × 10-1 = 0.34

7) 9 × 10^2 = 900

8) 6 × 10*0 = 6

Learn more about exponents on

https://brainly.com/question/13669161

#SPJ1


Related Questions

Emily has 6 pages of homework to do. If she can finish 38 of a page in one hour, how many hours will her homework take?

Answers

Emily's homework will take approximately 9 hours to complete.

Emily has 6 pages of homework and she can finish 3/8 of a page in one hour, we can calculate the total number of hours required to complete her homework.

To find the number of hours, we divide the total number of pages by the number of pages she can finish in one hour:

Number of hours = Total number of pages / Pages finished in one hour

Number of hours = 6 pages / (3/8) pages per hour

To divide by a fraction, we can multiply by its reciprocal:

Number of hours = 6 pages * (8/3) pages per hour

Simplifying the multiplication:

Number of hours = 48/3

Number of hours = 16

Therefore, Emily's homework will take approximately 16 hours to complete.

Hence, the answer is that Emily's homework will take approximately 9 hours to complete.

To know more about fraction , refer here:

https://brainly.com/question/10354322#

#SPJ11

Use spherical coordinates to evaluate the triple integral
∫∫∫E 4x^2 + 3dV = ______

Answers

The evaluation of the triple integral ∫∫∫E 4[tex]x^{2}[/tex] + 3dV is  (38/15)ππ

To evaluate the triple integral ∫∫∫E 4x^2 + 3dV in spherical coordinates, we need to express the integrand and the volume element dV in terms of the spherical coordinates ρ, θ, and φ.

The volume element dV in spherical coordinates is given by:
dV =  sin φ dρ dθ dφ
where ρ is the radial distance, θ is the azimuthal angle, and φ is the polar angle.
The region E in which we are integrating can be defined in spherical coordinates as follows:
0 ≤ ρ ≤ 2
0 ≤ θ ≤ 2π
0 ≤ φ ≤ π/2
Substituting these expressions into the volume element, we have:
dV =  sin φ dρ dθ dφ
= (sin φ) dρ dθ dφ
Now, we need to express the integrand 4[tex]x^2[/tex] + 3 in terms of the spherical coordinates.

The variable x can be expressed in terms of the spherical coordinates as:
x = ρ sin φ cos θ
Therefore, 4[tex]x^2[/tex] + 3 can be expressed as:
4[tex]x^2[/tex] + 3 = 4 [tex]sin^2[/tex] φ [tex]cos^2[/tex] θ + 3
Substituting this expression into the triple integral, we have:
∫∫∫E 4[tex]x^2[/tex] + 3dV
Now, we can evaluate the integral by performing the integration in the order φ, θ, ρ.
= (8/15)π + 2π
= (38/15)ππ

For similar questions on triple integral:

https://brainly.com/question/30404807

#SPJ11

A woman claims to have the ability to recognize by tasting it, whether tea was poured first and milk added after, or whether tea was added to milk. In order to test her powers, a set of 10 cups is brought to her and she is asked to taste them. She gets 7 out of 10 correct. Assuming each trial is independent, what is the probability that she would have done at least this well if she had no ability to recognize such difference

Answers

The probability that the woman would have done at least as well if she had no ability to recognize: the difference between the two methods is 0.117.

Let's assume that the woman has no ability to recognize the difference between the two methods. In that case, the probability of guessing the correct answer for each trial is 0.5 (since there are only two options).

The number of correct answers in 10 trials follows a binomial distribution with parameters n = 10 and p = 0.5. We want to calculate the probability of getting at least 7 correct answers.

Using a binomial distribution calculator or a standard normal distribution table, we can find that the probability of getting 7 or more correct answers is 0.117 (rounded to three decimal places).

Therefore, if the woman had no ability to recognize the difference between the two methods, there would still be a 0.117 probability that she would have gotten at least 7 correct answers by chance. Since 0.117 is not a small probability, we cannot reject the null hypothesis that the woman has no ability to recognize the difference between the two methods based solely on this experiment.

To know more about probability, refer here:

https://brainly.com/question/16383548#

#SPJ11

A person places $81200 in an investment account earning an annual rate of 3. 6%,


compounded continuously. Using the formula V = Pent, where Vis the value of the


account in tyears, P is the principal initially invested, e is the base of a natural


logarithm, and r is the rate of interest, determine the amount of money, to the


nearest cent, in the account after 13 years.

Answers

If a person places $81200 in an investment account earning an annual rate of 3. 6%, the amount of money in the account after 13 years is approximately $125689.60 to the nearest cent.

To solve this problem using the formula V = Pent, we need to plug in the given values.

P = $81200 (the principal initially invested)
r = 0.036 (the annual interest rate, expressed as a decimal)
t = 13 years

Using the formula V = Pent, we get:

V = $81200e^(0.036*13)

Using a calculator, we can evaluate e^(0.036*13) to be approximately 1.5498.

So V = $81200*1.5498 = $125689.60

Therefore, the amount of money in the account after 13 years is approximately $125689.60 to the nearest cent.

More on investment: https://brainly.com/question/28416628

#SPJ11

Sandeep's city took a telephone poll about a plan to build a new hotel downtown. 8,000 people took the poll. 92% of them were in favor of the new hotel. How many people were in favor of the new hotel?

Answers

Answer:  640

Step-by-step explanation:

The three inner circles are congruent

which measurement is closest to the

area of the largest outside circle in

square centimeters?

a 56. 52 cm

b 254. 34 cm

113 04 cm

5 cm

1,017 36 cm

Answers

The area of the largest outside circle in square centimeters is closest to e)1,017.36 cm².

The area of the largest circle is equal to the sum of the areas of the three inner circles and the area of the white region between them. Since the three inner circles are congruent, we can divide the white region into three equal parts. Let the radius of each inner circle be 'r'. Then, the radius of the largest circle is '3r'.

The area of the white region is the difference between the area of the square and the sum of the areas of the three congruent sectors. The area of each sector is (1/6)πr².

Therefore, the area of the white region is (9/4) r². Finally, we can use the formula for the area of a circle to find the area of the largest circle: A = π(3r)² + 3(1/6)πr² - (9/4) r² = (63/4)πr². If we substitute the value of r as 6 cm (since the diameter of the inner circle is 12 cm), we get the area of the largest circle as (63/4)π(6)² ≈ 1,017.36 cm²(e).

For more questions like Area click the link below:

https://brainly.com/question/27683633

#SPJ11

"Express the volume of the part of the ball p < 5 that lies between the cones т/4 and
т/3. "

Answers

We can express the limits of integration as follows:

For z between 0 and 5/√2, x and y range from 0 to √(25 - [tex]z^2[/tex]).

For z between 5/√2 and 5/2, x and y range from 0 to √(3[tex]z^2[/tex] - 25).

For z between 5/2 and 5, x and y range from 0 to √(25 - z

Find the equation of the sphere.

The equation of a sphere with center (0,0,0) and radius r is

[tex]x^2 + y^2 + z^2 = r^2.[/tex]

In this case, we have r = 5, so the equation of the sphere is

[tex]x^2 + y^2 + z^2 = 25.[/tex]

Find the equations of the cones.

The equation of a cone with half-aperture angle θ and vertex at the origin is given by [tex]x^2 + y^2 = z^2 tan^2[/tex](θ). In this case, we have two cones: one with θ = π/4 and one with θ = π/3.

Their equations are x^[tex]2 + y^2 = z^2 tan^2(\pi /4) = z^2[/tex] and [tex]x^2 + y^2 = z^2 tan^2(\pi /3) = 3z^2.[/tex]

Find the intersection points of the sphere and the cones.

To find the intersection points, we substitute the equation of the sphere into the equations of the cones: [tex]x^2 + y^2 + z^2 = 25, x^2 + y^2 = z^2,[/tex] and x^2 + [tex]y^2 = 3z^2[/tex]. This gives us two sets of equations:

[tex]x^2 + y^2 = z^2 and x^2 + y^2 + z^2 = 25:[/tex]

Substituting [tex]x^2 + y^2 = z^2[/tex] into[tex]x^2 + y^2 + z^2 = 25[/tex], we get [tex]2z^2 = 25[/tex],

which gives z = ±5/√2.

[tex]x^2 + y^2 = 3z^2 and x^2 + y^2 + z^2 = 25:[/tex]

Substituting[tex]x^2 + y^2 = 3z^2[/tex]into [tex]x^2 + y^2 + z^2 = 25[/tex], we get [tex]4z^2 = 25[/tex],

which gives z = ±5/2.

So we have four intersection points: (±5/√2, ±5/√2, ±5/√2) and (±5/2, ±5/2, ±5/2√3).

Find the part of the ball that lies between the cones.

To find the volume of the part of the ball that lies between the cones, we

need to integrate the volume element dV = dx dy dz over the region

enclosed by the cones and the sphere. Since the region is symmetric

about the z-axis, we can integrate over a quarter of the region and

multiply the result by 4.

for such more question on integration

https://brainly.com/question/22008756

#SPJ11

Question

Express the volume of the part of the ball that lies between two cones: one with a half-aperture angle of π/4 and the other with a half-aperture angle of π/3.

The Jones' family experienced a loss of $1,760 in purchasing power last year. If the inflation rate was
3%, find the percentage raise received on the family's $88,000 yearly income. Please explain

Answers

Thus, the percentage raise received on the family's $88,000 yearly income is 9.4%.

Explain about the percentage raise:

The difference in between final value and the starting value, stated as a percentage, is known as a percentage increase.

The base amount still determines whether a percentage rise or drop by a given percentage occurs. The absolute value change also changes if the basic amount does.

Hence, although the percentage rise or reduction is the same in this instance, the absolute increase is different.

Given data:

Yearly income = $88,000 Inflation rate - 3%

From the table, compound interest for the yearly inflation rate of 3% is 1.09417024.

Thus,

amount after compounding:

A = $88,000  * 1.09417024.

A = 96286.98112

A = $96286.98

percentage raise = (96286.98 - 88,000 )/ 88,000

percentage raise = 0.094 * 100

percentage raise = 9.4%

Thus, the percentage raise received on the family's $88,000 yearly income is 9.4%.

Know more about the percentage raise

https://brainly.com/question/11360390

#SPJ1

Nicole has 28 nickels and dimes that amount to $1. 85 how many of each coin does she have

Answers

Answer:

Nicole has 9 dimes and 19 nickels.

Nicole has 9 dimes and 19 nickels.

1.47 minutes is how many hours?
(1 hour = 60 minutes)

Answers

Answer :

1.47 Minutes = 0.0245 Hours.

Step-by-step explanation:

60 minutes = 1 hour

1 minute = 1/60

1 minute = 0.016666666666667 hours

1.47 minutes = 0.016666666666667 × 1.47

1.47 minute = 0.0245 hours

Therefore, 1.47 Minutes is equal to 0.0245 Hours.

Evaluate the following integral over the Region R. (Answer accurate to 2 decimal places). S 2x ) da 2(x + y) DA R R= {(x, y) | 9 < x² + y? < 49, x < 0} Hint: The integral and Region is defined in rectangular coordinates.

Answers

After integrating with respect to y and then x, we get the value of the integral accurate to 2 decimal places as -21.98.

First, let us express the limits of integration. Since the region R is defined in the rectangular coordinate system, we can express the limits of integration as follows:

9 < x² + y² < 49

-3 < x < 0

Next, we need to express the integral in terms of these limits of integration. The integral of 2x over the region R can be expressed as:

∫∫R 2x dA = ∫-3⁰ ∫√(9-x²)√(49-x²) 2x dy dx = -21.98

Here, we have used the fact that the region R is defined as {(x, y) | 9 < x² + y² < 49, x < 0}.

The limits of integration for y are determined by the equation of the circle centered at the origin with radius 7 and the equation of the circle centered at the origin with radius 3.

Now, we can evaluate the integral using the double integral formula.

To know more about integral here

https://brainly.com/question/18125359

#SPJ4

29 cm

L

8.5 cm

12 cm

Answers

Step-by-step explanation:

If u want to find the volume

V= width × length × height

V= 8.5 cm × 12 cm × 29 cm

V= 2958 cm3

Duncan's favorite park just added a statue of a badger, the state animal. The statue sits on a base shaped like a rectangular prism. The base is 5 feet long, 3 feet wide, and has a volume of 60 cubic feet. How tall is the base of the statue? Write your answer as a whole number or decimal. Do not round. PLEAS HELP â

LOL NVM

Answers

The height of the base of the statue, structured in rectangular prism shape with stated measure of dimension is 4 feet.

The volume of the rectangular prism will be given by the formula -

Volume = length × width × height

Keep the values in formula to find the value of height of the base of the statue

60 = 5 × 3 × height

Rearranging the equation in terms of height

Height = 60 × (5 × 3)

Multiplying the denominator on Right Hand Side

Height = 60/15

Divide the values

Height = 4

Hence the height is 4 feet.

Learn more about rectangular prism -

https://brainly.com/question/24284033

#SPJ4

Find Tn centered at x = 23 for all n for the function f(x) = ex. (Use symbolic notation and fractions where needed.)

Answers

For a function f(x) = e^x, we can find its Taylor series expansion Tn centered at x = 23 using the formula:

Tn(x) = Σ (f^(k)(23) * (x - 23)^k) / k!, for k = 0 to n

Since the derivative of e^x is always e^x, the k-th derivative evaluated at 23 is f^(k)(23) = e^23 for all k. Therefore, the Taylor series expansion becomes:

Tn(x) = Σ (e^23 * (x - 23)^k) / k!, for k = 0 to n

This is the Tn centered at x = 23 for all n for the function f(x) = e^x, with symbolic notation and fractions as requested.

To learn more about Taylor series, refer below:

https://brainly.com/question/29733106

#SPJ11

A coin (H: heads; T: tails) is flipped and a number cube (1, 2, 3, 4, 5, 6) is rolled. What is the sample space for this experiment?

Answers

The sample space for this experiment contains a total of 12 possible outcomes.

How to find the probability and determine the sample space?

The sample space for this experiment is the set of all possible outcomes. In this case, we have two independent events: flipping a coin and rolling a number cube.

The possible outcomes for flipping a coin are H (heads) and T (tails).

The possible outcomes for rolling a number cube are 1, 2, 3, 4, 5, and 6.

To determine the sample space for the experiment, we need to consider all possible combinations of these outcomes. Therefore, the sample space consists of all possible pairs of outcomes:

Sample space = {(H, 1), (H, 2), (H, 3), (H, 4), (H, 5), (H, 6), (T, 1), (T, 2), (T, 3), (T, 4), (T, 5), (T, 6)}

So the sample space for this experiment contains a total of 12 possible outcomes.

Learn more about probability and sample space.

brainly.com/question/11666439

#SPJ11

An investor purchases 500 shares of Exxon-mobil stock at $98. 93 per share. His broker charges 2% of the cost of the stock. What is the cost of the stock?

Answers

The cost of the stock, including the broker's fee, is $50,454.30.

How to find the total cost of stock?

The cost of the stock can be found by multiplying the number of shares purchased by the price per share. In this case, the investor purchased 500 shares of Exxon-mobil stock at $98.93 per share, so the cost of the stock can be calculated as follows:

Cost of stock = Number of shares × Price per share

Cost of stock = 500 × $98.93

Cost of stock = $49,465

However, the broker charges 2% of the cost of the stock, which is an additional fee that needs to be added to the total cost. To find the broker's fee, we can simply multiply the cost of the stock by 2%:

Broker's fee = 2% × Cost of stock

Broker's fee = 2% × $49,465

Broker's fee = $989.30

Therefore, the total cost of the stock, including the broker's fee, is:

Total cost of stock = Cost of stock + Broker's fee

Total cost of stock = $49,465 + $989.30

Total cost of stock = $50,454.30

Learn more about stocks

brainly.com/question/28663581

#SPJ11

7. The table shows the linear relationship between the total amount Mrs. Jacobs will be



charged for a skating party and the number of children attending.




Which equation best represents y, the total amount in dollars Mrs. Jacobs will be



charged for



x number of children attending the skating party?

Answers

Based on the information given in the table, we can see that there is a linear relationship between the total amount Mrs. Jacobs will be charged for a skating party and the number of children attending. This means that we can use a linear equation to represent this relationship.

To find the equation, we need to determine the slope (m) and y-intercept (b) of the line. We can do this by using two points from the table: (10, 100) and (20, 180).

The slope (m) can be calculated using the formula:

m = (y2 - y1) / (x2 - x1)

Plugging in the values, we get:

m = (180 - 100) / (20 - 10) = 8

The y-intercept (b) can be found by plugging in one of the points and the slope into the equation:

y = mx + b

Using the point (10, 100) and the slope we just calculated, we get:

100 = 8(10) + b

Solving for b, we get:

b = 20

Therefore, the equation that best represents y, the total amount in dollars Mrs. Jacobs will be charged for x number of children attending the skating party, is:

y = 8x + 20

This equation shows that for every additional child that attends the skating party, Mrs. Jacobs will be charged an additional $8, and the initial cost of the party is $20.

To know more about equation refer here

https://brainly.in/question/47251605#

#SPJ11

The seventh-grade class is selling boxes of votive candles as a fundraiser. The first box purchased costs $14. 00, and each additional box costs $10. 0.


a. Is the relationship between the number of additional boxes of candles purchased and the total money spent linear?explain


b. An equation that relates the total money spent on boxes of candles, C, to the number of additional boxes purchased,b,is,


c. Using the equation from part (b), the total money spent by a person who bought 3 boxes of candles for the fundraiser would be $

Answers

No, the relationship between the number of additional boxes of candles purchased. C = 14.00 + 10.00b. The person would spend $34.00 on 3 boxes of candles.

a. No, the relationship between the number of additional boxes of candles purchased and the total money spent is not linear. This is because the cost of the first box is $14.00 and the cost of each additional box is $10.00.

A linear relationship implies that the change in the dependent variable (total money spent) is proportional to the change in the independent variable (number of additional boxes purchased), but in this case, the cost does not change at a constant rate.

b. The equation that relates the total money spent on boxes of candles, C, to the number of additional boxes purchased, b, is:

C = 14.00 + 10.00b

This equation takes into account the cost of the first box, which is $14.00, and the cost of each additional box, which is $10.00.

c. If someone buys 3 boxes of candles, they are purchasing 2 additional boxes (since the first box is already included in the $14.00). Using the equation from part (b), the total money spent by a person who bought 3 boxes of candles for the fundraiser would be:

C = 14.00 + 10.00(2) = $34.00

Therefore, the person would spend $34.00 on 3 boxes of candles.

To know more about money spent, refer here:

https://brainly.com/question/1542778#

#SPJ11

At one of new york’s traffic signals, if more than 17 cars are held up at the intersection, a traffic officer will intervene and direct the traffic. the hourly traffic pattern from 12:00 p.m. to 10:00 p.m. mimics the random numbers generated between 5 and 25. (this holds true if there are no external factors such as accidents or car breakdowns.) scenario hour number of cars held up at intersection a noon−1:00 p.m. 16 b 1:00−2:00 p.m. 24 c 2:00−3:00 p.m. 6 d 3:00−4:00 p.m. 21 e 4:00−5:00 p.m. 15 f 5:00−6:00 p.m. 24 g 6:00−7:00 p.m. 9 h 7:00−8:00 p.m. 9 i 8:00−9:00 p.m. 9 based on the data in the table, what is the random variable in this scenario? a. the time interval between two red lights b. the number of traffic accidents that occur at the intersection c. the number of times a traffic officer monitors the signal d. the number of cars held up at the intersection

Answers

The random variable in this scenario is the number of cars held up at the intersection (option d).

The data provided in the table shows the number of cars held up at the intersection during specific time intervals, ranging from 12:00 p.m. to 9:00 p.m. Based on this information, it is clear that the random variable in this scenario is the number of cars held up at the intersection.

To put it in mathematical terms, let X be the random variable representing the number of cars held up at the intersection during a specific time interval. The data provided in the table represents a sample of X, with each time interval being a different observation. The values of X can range from 0 to 25, with 17 being the threshold for intervention by a traffic officer.

Therefore, the answer to the question is d. the number of cars held up at the intersection. It is important to note that this random variable is discrete, as it takes on specific integer values.

To know more about random variable here

https://brainly.com/question/17238189

#SPJ4

John bought stock for $350. A year later, he sold it for $385. What is his gain in dollars? What is his return on investment? (Round to the nearest whole percent. ) I need help, please

Answers

If John bought stock for $350 then A year later, he sold it for $385. So John's Return on investment is 10%.

To find John's gain in dollars, we need to subtract the purchase price from the selling price i.e. Gain = Selling price - The purchase price. So John's Return on investment is 10%.

Gain = $385 - $350

Gain = $35

So John's gain in dollars is $35.

To find John's return on investment (ROI), we need to use the formula:

ROI = (Gain / Investment) x 100%

We already know the gain is $35, and the investment is $350. Substituting these values into the formula, we get:

ROI = ($35 / $350) x 100%

ROI = 0.1 x 100%

ROI = 10%

So John's Return on investment is 10%. Rounded to the nearest whole percent, the answer is also 10%.

To learn more about “return on investment” refer to the https://brainly.com/question/11913993

#SPJ11

Write the equation for the translation of the graph of y = x + 7 one unit to the left.

Answers

Answer:

y=x+8

Step-by-step explanation:

since you are starting with the linear function y=x+7

a translation one unit to the left would be y=x+7(+1)

which gives us the answer y=x+8

The distance traveled by a car based on time, t, in seconds is given by the function of d =3t+1.
identify the independent variable, the dependent variable, rate of change and the initial value

Answers

The independent variable is 't' and the dependent variable is 'd' while the rate of change is '3' and the initial value is '1'.

In the given function d=3t+1:

The independent variable here is 't' which represents time in seconds.

whereas the dependent variable is 'd' and it represent the distance traveled by a car in some unit. The rate of change is 3, and it represents the constant speed of the car in units of distance per unit of time.

While the initial value is 1, which represents the distance traveled by the car when time is 0.

Hence this function can be interpreted as if the car is travelling at a constant speed of 3 units of distance and has already travelled 1 unit of distance at the start of the journey.

To know more about  independent variable and dependent variable:

https://brainly.com/question/13397927

#SPJ4

A sheet of paper 82 cm-by-88 cm is made into an open box (i.e. there's no top), by cutting X-cm squares out of each corner and folding up the sides. Find the value of x that maximizes the volume of the box. Give your answer in the simplified radical form. X= is the max.

Answers

The value of x that maximizes the volume of the box is x=11 cm.

Let x be the length of the side of each square cut from the corners of the paper.

The height of the box will be x cm, and the length and width of the base of the box will be (88-2x) cm and (82-2x) cm, respectively.

The volume of the box is given by V(x) = x(88-2x)(82-2x).

Expanding this expression and simplifying, we get V(x) = 4x^3 - 340x^2 + 7040x.

To find the maximum volume, we take the derivative of V(x) with respect to x and set it equal to 0. We get dV/dx = 12x^2 - 680x + 7040 = 0.

Solving this quadratic equation using the quadratic formula, we get x = (680 ± sqrt(680^2 - 4127040))/(2*12).

Simplifying this expression, we get x = (680 ± 120)/24.

Therefore, the two possible values of x are x = 25/3 cm and x = 11 cm.

To determine which value of x maximizes the volume of the box, we evaluate V(x) at both values of x and compare them. We find that V(25/3) ≈ 5757.04 cm^3 and V(11) = 5808 cm^3.

Therefore, the value of x that maximizes the volume of the box is x = 11 cm.

For more questions like Equation click the link below:

https://brainly.com/question/29657983

#SPJ11

At the beginning of spring, jin planted a small sunflower in his backyard. the sunflower's height in inches, h after w weeks, is given by the equation h=18+2.5w. what could the number 18 represent in the equation

Answers

After considering all the given data provided by the question we conclude that the number 18 present in the given equation  is  constant .


The equation for the sunflower's height is h = 18 + 2.5w.
Here, the number 18 shows the height of the sunflower during the time it was planted, so it is the constant term present in the equation and hence it does not relie on the duration of weeks that have passed.
The coefficient of w is 2.5, which represents the rate at which the sunflower grows in height per week.
Therefore, after one week, the sunflower would be 20.5 inches tall (18 + 2.5 x 1), after two weeks it would be 23 inches tall (18 + 2.5 x 2), after four weeks it would be 28 inches tall (18 + 2.5 x 4), and after six weeks it would be 33 inches tall (18 + 2.5 x 6).

To learn more about constant
https://brainly.com/question/27983400
#SPJ4

A ship sailed from Port X to Port Y. It traveled 20 kilometers due north and then 25 kilometers due west. If the ship then sailed back using the shortest route, what would the total distance traveled be? Round to the nearest kilometer.

Answers

The total distance traveled by the ship, including the trip from Port X to Port Y and the return trip, is approximately 42 kilometers.

What is Kilometer ?

Kilometer (km) is a metric unit of length or distance, commonly used in many countries around the world. It is equal to 1000 meters, or approximately 0.62 miles.

To find the shortest route back to Port X from Port Y, the ship needs to sail in a straight line. This means that it needs to sail due south for 20 kilometers and then due east for 25 kilometers.

We can now use the Pythagorean theorem to find the total distance traveled by the ship:

total distance = √(400+ 625 + 400+ 625)

total distance = √(1200 + 625)

total distance = √1825

total distance ≈ 42.73 kilometers (rounded to the nearest kilometer)

Therefore, the total distance traveled by the ship, including the trip from Port X to Port Y and the return trip, is approximately 42 kilometers.

To learn more about Kilometer from given link.

https://brainly.com/question/22040991

#SPJ1

Decide whether the triangles are similar. If so, determine the appropriate expression to solve for x.

Triangles ABC and EDF; triangle ABC has angle A measuring 53 degrees, angle C measuring 62 degrees, side AC labeled as y, side AB labeled as w, and side BC labeled as x; triangle EDF has angle D measuring 61 degrees, angle F measuring 53 degrees, side DE labeled z, side EF labeled u, and side DF labeled r.

The triangles are not similar; no expression for x can be found.
ΔABC ~ ΔDEF; x equals r times w over u
ΔABC ~ ΔEFD; x equals r times w over u
ΔABC ~ ΔEFD; x equals r times w over z

Answers

The triangles ABC and EDF are similar, and x = r × w/u.

As per the question, we have angle A in triangle ABC congruent to angle F in triangle EDF, angle C in triangle ABC congruent to angle D in triangle EDF, and angle B in triangle ABC congruent to angle E in triangle EDF.

Therefore, the triangles are similar by the Angle-Angle (AA) similarity theorem.

To find the expression for x, we can use the fact that the corresponding sides of similar triangles are proportional.

In this case, we have:

x/w = u/r (corresponding sides of similar triangles)

Solving for x, we get:

x = r × w/u

Therefore, x = r × w/u, and it can use this expression to solve for x in triangle ABC and EDF.

Learn more about similar triangles here:

brainly.com/question/25882965

#SPJ1

Please help it would be amazing if you knew this

Answers

Answer: 5x + 5

Step-by-step explanation:

You combine the two functions togther, and add the like terms.

2x +3 +3x +2

Please give brainliest, have a great night!

Determine whether the series n² - 5 na tn - 6 n=1 is convergent or divergent using the Limit Comparison Test.

Answers

To use the Limit Comparison Test, we need to find a series whose behavior is well-known and similar to the given series. Let's consider the series aₙ = n². We have:

limₙ→∞ (aₙ / (n² - 5naₙ - 6)) = limₙ→∞ (n² / n²) = 1

Since this limit is finite and positive, and aₙ is a convergent series (by the p-series test with p = 2), we can apply the Limit Comparison Test and conclude that the given series is convergent.
To determine if the series ∑(n² - 5n) from n=1 to infinity is convergent or divergent using the Limit Comparison Test, we need to find a comparable series and then calculate the limit of the ratio between the two series as n approaches infinity.

Let's compare the given series to a simpler series ∑n² (n=1 to infinity). Now, we'll find the limit of the ratio:

Limit (n→∞) [(n² - 5n) / n²]

As n approaches infinity, the -5n term becomes insignificant compared to the n² term. So, the limit becomes:

Limit (n→∞) [n² / n²] = 1

Since the limit is a finite, nonzero value (1 in this case), the given series and the comparison series will have the same convergence behavior. We know that the series ∑n² (n=1 to infinity) is a divergent series, as it is a p-series with p=2 (less than or equal to 1). Therefore, the given series ∑(n² - 5n) from n=1 to infinity is also divergent.

Learn more about Limit Comparison Test here: brainly.com/question/31362838

#SPJ11

5-|p+6|=-8

2 answers

NOT 19

Answers

5-|p+6|=-8
-|p+6|=-13
|p+6|=13
-> two equations —-
p+6=13
p+6=-13
(Absolute value means that no matter what is in it it will be positive thus if it equals +-13 it is a valid solution)
—-p=7,-19

please please please please please please help me this is all due tomorrow​

Answers

For the following probabilities:

7. Theoretically, blue will occur 100 times.8. Based on experiment, blue will occur 95-105 times.9. a) 1/4, b) 1/2, c) 3/4.10. a) 0.25, b) 0.5, c) 0.75.11. spade can occur 125 times theoretically.12. experimentally spade occurs 500 times.

How to determine probability?

7. Theoretically, if the spinner is spun 400 times, you would expect to get blue 100 times since blue has a probability of 1/4 or 25% of being selected on each spin.

8. Based on the experiment, if the spinner is spun 400 times, you would expect to get blue around 95-105 times, depending on the margin of error in the experiment. This is based on the observed experimental probability of blue being selected in the given number of spins.

9. a) P(club) = 13/52 or 1/4

b) P(red card) = 26/52 or 1/2

c) P(not a heart) = 39/52 or 3/4

10. a) P(club) = 5/30 or 1/6 in the experiment, which is close to the theoretical probability of 1/4 or 0.25.

b) P(red card) = 13/30 in the experiment, which is close to the theoretical probability of 1/2 or 0.5.

c) P(not a heart) = 27/30 in the experiment, which is close to the theoretical probability of 3/4 or 0.75.

11. Theoretically, if a card is drawn at random 500 times, you would expect to get a spade around 125 times since spades have a probability of 1/4 or 25% of being selected on each draw.

12. Based on the experiment, if a card is drawn at random 500 times, you would expect to get a spade around 110-140 times, depending on the margin of error in the experiment. This is based on the observed experimental probability of spades being selected in the given number of draws.

Find out more on probability here: https://brainly.com/question/251701

#SPJ1

Image transcribed:

7. Theoretically, if the spinner is spun 400 times, how many times would you expect to get blue?

8. Based on the experiment, if the spinner is spun 400 times, how many times would you expect to get blue?

9. A card is drawn from a standard deck of cards. Find each probability.

a) P(club)

b) P(red card)

c) P(not a heart)

10. The table below shows the results of an experiment in which a card was drawn at random 30 times. Find each probability based on the experiment and compare to the theoretical probability.

Result | Frequency

Heart | 3

Diamond | 10

Club | 5

Spade | 12

a) P(club)

b) P(red card)

c) P(not a heart)

11. Theoretically, if a card is drawn at random 500 times, how many times would you expect to get a spade?

12. Based on the experiment, if a card is drawn at random 500 times. how many times would you expect to get a spade?

Other Questions
In Passage 2, what effect does the repetition of the phrase if you have ever woken up throughout the poem have on the poems message? What would be the linear velocity of a boy's toes doing a cartwheel who is 2.1 m long from the tip of his toes to the end of his fingers and who is experiencing a centripetal force of 5.0 m/s2? What was the greatest challenge and obstacle for the archaeologist that found the pompeii? Alguien q pueda explicar esto The cafeteria staff made sandwiches. Each sandwich had either rye or white bread, either ham or turkey, and either cheese or no cheese. The staff made an equal number of each type of sandwich. The sandwiches were placed on a tray. Without looking, Mary will choose a sandwich. What are the chances that Mary will get a sandwich with cheese?Responsesone eighthone sixthone thirdone half Objective: Use the different forms of energy to generate heat You are stranded at a cabin in Alaska in winter. Will you stay warm? You need to find 6 ways to create heat. The cabin has modern appliances and has working power, but the furnace is broken. The cabin has a garage full of equipment, including a car (but there's too much snow to get it out of the garage), an electric lawnmower, a fireplace, and some kindling. Good news! There's a computer and an internet connection inside the cabin tan(x)>(3^1/2)+1 Range for x( 0 Which type of figurative language appeals to the five senses?metaphorssimilesallusionssensory details Which expression is equivalent to 3^4? This circle is centered at the origin, and the length of its radius is 5. What is the equation of the circle? A. x2 + y2 = 52 B. x2 + y2 = 5 C. (x - 5)2 + (y - 5)2 = 25 D. Describe the different types of sports under its categories and its nature. Environmental scientists can use a similar lab kit to test collected water samples frombodies of water. In lakes and streams, calcium carbonate (CaCO3) causes alkalinity,which allows it to function as a buffer, neutralizing any acid rain that may enter thewater supply. A buffer is a substance that serves to resist small changes in acidity oralkalinity in a solution. Environmental scientists monitoring pollution levels are measuring buffer levels intwo specific lakes. They found that Lake B had a greater ppm of calcium carbonatethan Lake A. Which of the two lakes would be able to neutralize a greater amount of acid rain?Explain your answer. For f(x)=1/x^2 show there is no c such that f(1)-f(-1)=f'(c)(2).Explain why the mean value theorem doesnt apply over the interval[-1,1]" write a subroutine called create employee that accepts two string parameters for the new name and title and one integer parameter for the id. it should return a newly allocated employee structure with all of the fields filled in. g Your antique watch is increasing in value at a rate of 5% each year. If it is worth $500 today, how much will it be worth 3 years from now? Round to the nearest hundredths. A number cube has sides numbered 1 through 6. the probability of rolling a 2 is 1/6 what is the probability of not rolling a 2? enter your answer as a fraction, in simplest form, in the box. a calculator is allowed on this quiz.question 1 options:56162376 What is the coefficient in front of Cl, when this equation is balanced?Zn +_Cl ZnCl PQRSTU is plotted on a coordinate plane with vertices p(-1,0) Q(-1,3) R(1,4) S(2,5) T(4,4) and U(4,0) what is the hexagons area Let u = - 3i + 2j, v = 3i - 3j and w = - 3jFind the specified scalar or vector.5u(3v - 4w) Heroes of the WildStudy the interactive in the lesson to describe the background and contributions of these key historical figures. Then respond to the reflection question. Your answers should be written in complete sentences.LewisClarkSacajaweaYorkDescribe eachindividuals background.Describe thecontributions he/she made to the expedition.Reflection QuestionWhy was it important to send explorers into the new territory of the United States?